Series expansion of $logfrac{sin z}{z}$












1












$begingroup$


What is the series expansion of $f(z)= logfrac{sin z}{z}$ upto $z^6$ terms?
Since $f(z)$ is an even function,so the coefficients of all the odd powers of z must be $0$ and I calculated the coefficient of $z^2 $ to be $-1/4$ (am I correct) but what will be the expansion till $z^6$?










share|cite|improve this question











$endgroup$

















    1












    $begingroup$


    What is the series expansion of $f(z)= logfrac{sin z}{z}$ upto $z^6$ terms?
    Since $f(z)$ is an even function,so the coefficients of all the odd powers of z must be $0$ and I calculated the coefficient of $z^2 $ to be $-1/4$ (am I correct) but what will be the expansion till $z^6$?










    share|cite|improve this question











    $endgroup$















      1












      1








      1


      1



      $begingroup$


      What is the series expansion of $f(z)= logfrac{sin z}{z}$ upto $z^6$ terms?
      Since $f(z)$ is an even function,so the coefficients of all the odd powers of z must be $0$ and I calculated the coefficient of $z^2 $ to be $-1/4$ (am I correct) but what will be the expansion till $z^6$?










      share|cite|improve this question











      $endgroup$




      What is the series expansion of $f(z)= logfrac{sin z}{z}$ upto $z^6$ terms?
      Since $f(z)$ is an even function,so the coefficients of all the odd powers of z must be $0$ and I calculated the coefficient of $z^2 $ to be $-1/4$ (am I correct) but what will be the expansion till $z^6$?







      complex-analysis power-series






      share|cite|improve this question















      share|cite|improve this question













      share|cite|improve this question




      share|cite|improve this question








      edited Jan 12 at 9:42









      Kemono Chen

      3,0721743




      3,0721743










      asked Jan 12 at 9:03









      Nilesh KhatriNilesh Khatri

      195




      195






















          2 Answers
          2






          active

          oldest

          votes


















          2












          $begingroup$

          Using steps, start with
          $$log left(frac{sin (x)}{x}right)=log left(1+left(frac{sin (x)}{x}-1right)right)$$ Let $t=frac{sin (x)}{x}-1$ and use
          $$log(1+t)=t-frac{t^2}{2}+frac{t^3}{3}-frac{t^4}{4}+frac{t^5}{5}-frac{t^6}{6}+frac{t^7}{
          7}+Oleft(t^8right)$$

          Now
          $$t=frac{sin (x)}{x}-1=-frac{x^2}{6}+frac{x^4}{120}-frac{x^6}{5040}+Oleft(x^8right)$$ I suppose that the binomila expansion will be simple since you are not asked for too many terms.






          share|cite|improve this answer









          $endgroup$





















            3












            $begingroup$

            I will post another approach with Bernoulli numbers. This answer provides the general terms of the coefficients.
            $$lnfrac{sin x}x=int_0^xleft(cot t-frac1tright)dt\
            =int_0^xsum_{nge 1}frac{(-4)^nB_{2n}}{(2n)!}t^{2n-1}dt\
            =sum_{nge 1}frac{(-4)^nB_{2n}}{(2n)!(2n)}x^{2n}\
            =-frac16x^2-frac1{180}x^4-frac1{2835}x^6-cdots$$






            share|cite|improve this answer









            $endgroup$













            • $begingroup$
              Nice way to do it for sure ! $to +1$
              $endgroup$
              – Claude Leibovici
              Jan 12 at 10:23










            • $begingroup$
              This is a really good method.
              $endgroup$
              – Szeto
              Jan 13 at 2:59











            Your Answer





            StackExchange.ifUsing("editor", function () {
            return StackExchange.using("mathjaxEditing", function () {
            StackExchange.MarkdownEditor.creationCallbacks.add(function (editor, postfix) {
            StackExchange.mathjaxEditing.prepareWmdForMathJax(editor, postfix, [["$", "$"], ["\\(","\\)"]]);
            });
            });
            }, "mathjax-editing");

            StackExchange.ready(function() {
            var channelOptions = {
            tags: "".split(" "),
            id: "69"
            };
            initTagRenderer("".split(" "), "".split(" "), channelOptions);

            StackExchange.using("externalEditor", function() {
            // Have to fire editor after snippets, if snippets enabled
            if (StackExchange.settings.snippets.snippetsEnabled) {
            StackExchange.using("snippets", function() {
            createEditor();
            });
            }
            else {
            createEditor();
            }
            });

            function createEditor() {
            StackExchange.prepareEditor({
            heartbeatType: 'answer',
            autoActivateHeartbeat: false,
            convertImagesToLinks: true,
            noModals: true,
            showLowRepImageUploadWarning: true,
            reputationToPostImages: 10,
            bindNavPrevention: true,
            postfix: "",
            imageUploader: {
            brandingHtml: "Powered by u003ca class="icon-imgur-white" href="https://imgur.com/"u003eu003c/au003e",
            contentPolicyHtml: "User contributions licensed under u003ca href="https://creativecommons.org/licenses/by-sa/3.0/"u003ecc by-sa 3.0 with attribution requiredu003c/au003e u003ca href="https://stackoverflow.com/legal/content-policy"u003e(content policy)u003c/au003e",
            allowUrls: true
            },
            noCode: true, onDemand: true,
            discardSelector: ".discard-answer"
            ,immediatelyShowMarkdownHelp:true
            });


            }
            });














            draft saved

            draft discarded


















            StackExchange.ready(
            function () {
            StackExchange.openid.initPostLogin('.new-post-login', 'https%3a%2f%2fmath.stackexchange.com%2fquestions%2f3070718%2fseries-expansion-of-log-frac-sin-zz%23new-answer', 'question_page');
            }
            );

            Post as a guest















            Required, but never shown

























            2 Answers
            2






            active

            oldest

            votes








            2 Answers
            2






            active

            oldest

            votes









            active

            oldest

            votes






            active

            oldest

            votes









            2












            $begingroup$

            Using steps, start with
            $$log left(frac{sin (x)}{x}right)=log left(1+left(frac{sin (x)}{x}-1right)right)$$ Let $t=frac{sin (x)}{x}-1$ and use
            $$log(1+t)=t-frac{t^2}{2}+frac{t^3}{3}-frac{t^4}{4}+frac{t^5}{5}-frac{t^6}{6}+frac{t^7}{
            7}+Oleft(t^8right)$$

            Now
            $$t=frac{sin (x)}{x}-1=-frac{x^2}{6}+frac{x^4}{120}-frac{x^6}{5040}+Oleft(x^8right)$$ I suppose that the binomila expansion will be simple since you are not asked for too many terms.






            share|cite|improve this answer









            $endgroup$


















              2












              $begingroup$

              Using steps, start with
              $$log left(frac{sin (x)}{x}right)=log left(1+left(frac{sin (x)}{x}-1right)right)$$ Let $t=frac{sin (x)}{x}-1$ and use
              $$log(1+t)=t-frac{t^2}{2}+frac{t^3}{3}-frac{t^4}{4}+frac{t^5}{5}-frac{t^6}{6}+frac{t^7}{
              7}+Oleft(t^8right)$$

              Now
              $$t=frac{sin (x)}{x}-1=-frac{x^2}{6}+frac{x^4}{120}-frac{x^6}{5040}+Oleft(x^8right)$$ I suppose that the binomila expansion will be simple since you are not asked for too many terms.






              share|cite|improve this answer









              $endgroup$
















                2












                2








                2





                $begingroup$

                Using steps, start with
                $$log left(frac{sin (x)}{x}right)=log left(1+left(frac{sin (x)}{x}-1right)right)$$ Let $t=frac{sin (x)}{x}-1$ and use
                $$log(1+t)=t-frac{t^2}{2}+frac{t^3}{3}-frac{t^4}{4}+frac{t^5}{5}-frac{t^6}{6}+frac{t^7}{
                7}+Oleft(t^8right)$$

                Now
                $$t=frac{sin (x)}{x}-1=-frac{x^2}{6}+frac{x^4}{120}-frac{x^6}{5040}+Oleft(x^8right)$$ I suppose that the binomila expansion will be simple since you are not asked for too many terms.






                share|cite|improve this answer









                $endgroup$



                Using steps, start with
                $$log left(frac{sin (x)}{x}right)=log left(1+left(frac{sin (x)}{x}-1right)right)$$ Let $t=frac{sin (x)}{x}-1$ and use
                $$log(1+t)=t-frac{t^2}{2}+frac{t^3}{3}-frac{t^4}{4}+frac{t^5}{5}-frac{t^6}{6}+frac{t^7}{
                7}+Oleft(t^8right)$$

                Now
                $$t=frac{sin (x)}{x}-1=-frac{x^2}{6}+frac{x^4}{120}-frac{x^6}{5040}+Oleft(x^8right)$$ I suppose that the binomila expansion will be simple since you are not asked for too many terms.







                share|cite|improve this answer












                share|cite|improve this answer



                share|cite|improve this answer










                answered Jan 12 at 9:14









                Claude LeiboviciClaude Leibovici

                121k1157133




                121k1157133























                    3












                    $begingroup$

                    I will post another approach with Bernoulli numbers. This answer provides the general terms of the coefficients.
                    $$lnfrac{sin x}x=int_0^xleft(cot t-frac1tright)dt\
                    =int_0^xsum_{nge 1}frac{(-4)^nB_{2n}}{(2n)!}t^{2n-1}dt\
                    =sum_{nge 1}frac{(-4)^nB_{2n}}{(2n)!(2n)}x^{2n}\
                    =-frac16x^2-frac1{180}x^4-frac1{2835}x^6-cdots$$






                    share|cite|improve this answer









                    $endgroup$













                    • $begingroup$
                      Nice way to do it for sure ! $to +1$
                      $endgroup$
                      – Claude Leibovici
                      Jan 12 at 10:23










                    • $begingroup$
                      This is a really good method.
                      $endgroup$
                      – Szeto
                      Jan 13 at 2:59
















                    3












                    $begingroup$

                    I will post another approach with Bernoulli numbers. This answer provides the general terms of the coefficients.
                    $$lnfrac{sin x}x=int_0^xleft(cot t-frac1tright)dt\
                    =int_0^xsum_{nge 1}frac{(-4)^nB_{2n}}{(2n)!}t^{2n-1}dt\
                    =sum_{nge 1}frac{(-4)^nB_{2n}}{(2n)!(2n)}x^{2n}\
                    =-frac16x^2-frac1{180}x^4-frac1{2835}x^6-cdots$$






                    share|cite|improve this answer









                    $endgroup$













                    • $begingroup$
                      Nice way to do it for sure ! $to +1$
                      $endgroup$
                      – Claude Leibovici
                      Jan 12 at 10:23










                    • $begingroup$
                      This is a really good method.
                      $endgroup$
                      – Szeto
                      Jan 13 at 2:59














                    3












                    3








                    3





                    $begingroup$

                    I will post another approach with Bernoulli numbers. This answer provides the general terms of the coefficients.
                    $$lnfrac{sin x}x=int_0^xleft(cot t-frac1tright)dt\
                    =int_0^xsum_{nge 1}frac{(-4)^nB_{2n}}{(2n)!}t^{2n-1}dt\
                    =sum_{nge 1}frac{(-4)^nB_{2n}}{(2n)!(2n)}x^{2n}\
                    =-frac16x^2-frac1{180}x^4-frac1{2835}x^6-cdots$$






                    share|cite|improve this answer









                    $endgroup$



                    I will post another approach with Bernoulli numbers. This answer provides the general terms of the coefficients.
                    $$lnfrac{sin x}x=int_0^xleft(cot t-frac1tright)dt\
                    =int_0^xsum_{nge 1}frac{(-4)^nB_{2n}}{(2n)!}t^{2n-1}dt\
                    =sum_{nge 1}frac{(-4)^nB_{2n}}{(2n)!(2n)}x^{2n}\
                    =-frac16x^2-frac1{180}x^4-frac1{2835}x^6-cdots$$







                    share|cite|improve this answer












                    share|cite|improve this answer



                    share|cite|improve this answer










                    answered Jan 12 at 9:48









                    Kemono ChenKemono Chen

                    3,0721743




                    3,0721743












                    • $begingroup$
                      Nice way to do it for sure ! $to +1$
                      $endgroup$
                      – Claude Leibovici
                      Jan 12 at 10:23










                    • $begingroup$
                      This is a really good method.
                      $endgroup$
                      – Szeto
                      Jan 13 at 2:59


















                    • $begingroup$
                      Nice way to do it for sure ! $to +1$
                      $endgroup$
                      – Claude Leibovici
                      Jan 12 at 10:23










                    • $begingroup$
                      This is a really good method.
                      $endgroup$
                      – Szeto
                      Jan 13 at 2:59
















                    $begingroup$
                    Nice way to do it for sure ! $to +1$
                    $endgroup$
                    – Claude Leibovici
                    Jan 12 at 10:23




                    $begingroup$
                    Nice way to do it for sure ! $to +1$
                    $endgroup$
                    – Claude Leibovici
                    Jan 12 at 10:23












                    $begingroup$
                    This is a really good method.
                    $endgroup$
                    – Szeto
                    Jan 13 at 2:59




                    $begingroup$
                    This is a really good method.
                    $endgroup$
                    – Szeto
                    Jan 13 at 2:59


















                    draft saved

                    draft discarded




















































                    Thanks for contributing an answer to Mathematics Stack Exchange!


                    • Please be sure to answer the question. Provide details and share your research!

                    But avoid



                    • Asking for help, clarification, or responding to other answers.

                    • Making statements based on opinion; back them up with references or personal experience.


                    Use MathJax to format equations. MathJax reference.


                    To learn more, see our tips on writing great answers.




                    draft saved


                    draft discarded














                    StackExchange.ready(
                    function () {
                    StackExchange.openid.initPostLogin('.new-post-login', 'https%3a%2f%2fmath.stackexchange.com%2fquestions%2f3070718%2fseries-expansion-of-log-frac-sin-zz%23new-answer', 'question_page');
                    }
                    );

                    Post as a guest















                    Required, but never shown





















































                    Required, but never shown














                    Required, but never shown












                    Required, but never shown







                    Required, but never shown

































                    Required, but never shown














                    Required, but never shown












                    Required, but never shown







                    Required, but never shown







                    Popular posts from this blog

                    Can a sorcerer learn a 5th-level spell early by creating spell slots using the Font of Magic feature?

                    Does disintegrating a polymorphed enemy still kill it after the 2018 errata?

                    A Topological Invariant for $pi_3(U(n))$